Search results

Page title matches

Page text matches

  • ...ments. Let <math>S</math> be a set of positive integers, none greater than 15. Suppose no two disjoint subsets of <math>S</math> have the same sum. What ...> must have more than 4 elements, otherwise its sum would be at most <math>15+14+13+12=54</math>.
    2 KB (364 words) - 19:41, 1 September 2020
  • ...ath>1</math>). By the [[Binomial Theorem]], this is <math>(-1) \cdot (-1)^{15}{18 \choose 3} = \boxed{816}</math>. ...y^2</math> term is <math>{2\choose 0} + {3\choose 1} + \cdots + {17\choose 15}</math>. This is actually the sum of the first 16 triangular numbers, which
    6 KB (872 words) - 16:51, 9 June 2023
  • ...ath>a+b+c=\frac{3194+m}{222}>\frac{3194}{222}>14</math> so <math>a+b+c\geq 15</math>. Checking each of the multiples of <math>222</math> from <math>15\cdot222</math> to <math>18\cdot222</math> by subtracting <math>N</math> fro
    3 KB (565 words) - 16:51, 1 October 2023
  • By similar triangles, <math>BE'=\frac{d}{510}\cdot450=\frac{15}{17}d</math> and <math>EC=\frac{d}{425}\cdot450=\frac{18}{17}d</math>. Sinc ...510 - \frac{6}{5}d + 510 - \frac{17}{15}d \Longrightarrow d\left(\frac{50}{15}\right) = 1020 \Longrightarrow d = \boxed{306}</math>.
    11 KB (1,850 words) - 18:07, 11 October 2023
  • ...\log 1 + \log 2 + \log 4 + \log 5 +\ldots + \log 1000000 = \log (2^05^0)(2^15^0)(2^25^0)\cdots (2^65^6).</cmath> Each power of <math>2</math> appears <ma
    3 KB (487 words) - 20:52, 16 September 2020
  • ...right)^2}{2\cdot 2\sqrt5\cdot2\sqrt6}=\frac{16}{8\sqrt{30}}=\sqrt{\frac{2}{15}},</cmath> so <math>\sin\theta=\sqrt{1-\cos^2\theta}=\sqrt{\frac{13}{15}}.</math>
    3 KB (460 words) - 00:44, 5 February 2022
  • {{AIME box|year=1987|num-b=13|num-a=15}}
    7 KB (965 words) - 10:42, 12 April 2024
  • ...ance at rate <math>r</math> from the escalator, while Bob is getting <math>15</math> seconds of help at rate <math>r</math>. Solving for <math>r</math>,
    7 KB (1,187 words) - 16:21, 27 January 2024
  • ...or which there is a unique integer <math>k</math> such that <math>\frac{8}{15} < \frac{n}{n + k} < \frac{7}{13}</math>? <cmath>\begin{array}{ccccc}\frac{15}{8} &>& \frac{k + n}{n} &>& \frac{13}{7}\\
    2 KB (393 words) - 16:59, 16 December 2020
  • ...h>|x - 60| = 0</math>, which is when <math>x = 60</math> and <math>y = \pm 15</math>. Since the graph is [[symmetry|symmetric]] about the y-axis, we just ...using the [[Shoelace Theorem]], we get <math>2 \cdot \frac{1}{2}(80 - 48)(15) = \boxed{480}</math>.
    2 KB (371 words) - 17:25, 13 February 2024
  • ...6,\ 2^3 = 8,\ 2 \cdot 5 = 10,</math> <math>\ 2 \cdot 7 = 14,\ 3 \cdot 5 = 15,\ 3 \cdot 7 = 21,</math> <math>\ 2 \cdot 11 = 22,\ 2 \cdot 13 = 26,</math>
    3 KB (511 words) - 09:29, 9 January 2023
  • {{AIME box|year=1988|num-b=13|num-a=15}}
    4 KB (700 words) - 17:21, 3 May 2021
  • ...+ F_{15}) + 1 &\Longrightarrow (aF_{17} + bF_{16})\cdot x + (aF_{16} + bF_{15} + 1) = 0,\ x\not\in Q \\ &\Longrightarrow aF_{17} + bF_{16} = 0 \text{ and } aF_{16} + bF_{15} + 1 = 0 \\
    10 KB (1,585 words) - 03:58, 1 May 2023
  • <math>504 = 3m + 15</math>
    2 KB (422 words) - 00:22, 6 September 2020
  • ...use the arithmetic progression from left to right has difference <math>x - 15</math>. Therefore, we have <math>x = 50</math>, and because the desired ast
    5 KB (878 words) - 23:06, 20 November 2023
  • ...ac{[APC] + [BPC]}{[APB]} = 3</math>, so <math>CP = \dfrac{3}{4} \cdot CF = 15</math>. ...ed out, so <math>w_C = 1</math> and <math>w_F = 3</math>. Thus, <math>CP = 15</math> and <math>PF = 5</math>.
    13 KB (2,091 words) - 00:20, 26 October 2023
  • {{AIME box|year=1989|num-b=13|num-a=15}}
    2 KB (408 words) - 17:28, 16 September 2023
  • <math>n = 4: 5000+15*116 = 6740</math>
    5 KB (851 words) - 18:01, 28 December 2022
  • label("$P$",(6,15),N); label("$X$",(12.5,15),N);
    6 KB (980 words) - 15:08, 14 May 2024
  • <math>m = 15</math> gives a solution for k. <math>10 + 5a = 15^3</math>
    3 KB (552 words) - 12:41, 3 March 2024

View (previous 20 | next 20) (20 | 50 | 100 | 250 | 500)